Ana içeriğe geç
Microsoft
|
Math Solver
Çözüm
Pratik
Oynamak
Başlıklar
Ön Cebir
Ortalama
Mod
En Büyük Ortak Faktör
En küçük ortak Kat
İşlemler Sırası
Kesirler
Karışık Kesirler
Asal Çarpanlara Ayırma
Katsayılar
Kökler
Cebir
Benzer Terimleri Birleştir
Bir Değişken için Çöz
Çarpan
Genişlet
Kesirleri Değerlendirin
Doğrusal Denklemler
Kuadratik Denklemler
Eşitsizlikler
Denklem Sistemleri
Matrisler
Trigonometri
Sadeleştir
Değerlendir
Grafikler
Denklemleri Çöz
Hesaplama
Türevler
İntegraller
Limitler
Cebir Girişleri
Trigonometri Girişleri
Matematik Girdileri
Matris Girişleri
Çözüm
Pratik
Oynamak
Başlıklar
Ön Cebir
Ortalama
Mod
En Büyük Ortak Faktör
En küçük ortak Kat
İşlemler Sırası
Kesirler
Karışık Kesirler
Asal Çarpanlara Ayırma
Katsayılar
Kökler
Cebir
Benzer Terimleri Birleştir
Bir Değişken için Çöz
Çarpan
Genişlet
Kesirleri Değerlendirin
Doğrusal Denklemler
Kuadratik Denklemler
Eşitsizlikler
Denklem Sistemleri
Matrisler
Trigonometri
Sadeleştir
Değerlendir
Grafikler
Denklemleri Çöz
Hesaplama
Türevler
İntegraller
Limitler
Cebir Girişleri
Trigonometri Girişleri
Matematik Girdileri
Matris Girişleri
Temel
Cebir
Trigonometri
hesap
İstatistikler
Matrisler
Karakter
Hesapla
\infty
Test
Limits
Şuna benzer 5 problem:
\lim_{ x \rightarrow 0 } \frac{1}{x^2}
Web Aramasından Benzer Problemler
Showing that the \lim_{x\to 0}\frac{1}{x^2} does not exist
https://math.stackexchange.com/q/1579837
Suppose that the limit exists and equals c\in\mathbb{R}. Then for e.g. \epsilon>1 some \delta>0 must exist with \left|x\right|<\delta\implies\left|\frac{1}{x^{2}}-c\right|<1. However, if we ...
Applying L'Hopital's rule to \lim\limits_{x \to 0}\frac{2}{x^2}
https://math.stackexchange.com/questions/502024/applying-lhopitals-rule-to-lim-limits-x-to-0-frac2x2
In order to use the 0/0 case of L'Hospital's rule, we require that both the numerator and the denominator tend to 0 at the appropriate point. The numerator does not tend to 0.
Is this piece-wise function continuous, and why?
https://math.stackexchange.com/questions/2411697/is-this-piece-wise-function-continuous-and-why
If we look at the behaviour as x approaches zero from the right, the function looks like this: \begin{matrix}x & f(x) = \frac{1}{x^2} \\ 1 & 1 \\ 0.1 & 100 \\ 0.01 & 10000 \\ 0.001 & 1000000 \\ 0.0001 & 100000000\end{matrix} ...
Manipulating \lim\limits_{x \to 0}{\frac{\sqrt{x+\sqrt{x}}}{x^n}}
https://math.stackexchange.com/questions/2177214/manipulating-lim-limits-x-to-0-frac-sqrtx-sqrtxxn
If \lim\limits_{x \to 0}{\frac{\sqrt{x+\sqrt{x}}}{x^n}} = c for some c\neq 0, then \lim\limits_{x \to 0}{\frac{x+\sqrt{x}}{x^{2n}}} =c^2. Now, let \sqrt{x}=t. We then wish to find n such ...
Limit of \frac{f'(x)}{g'(x)} & g'(x) \neq 0 in Hypotheses of L'Hospital's rule.
https://math.stackexchange.com/q/110408
When we write things like \lim_{x\to a}h(x) = \lim_{x\to a}H(x) we usually mean "if either limit exists, then they both do and they are equal; if either limit does not exist, then neither limit ...
How do we calculate the Right and Left Hand Limit of 1/x?
https://math.stackexchange.com/questions/762599/how-do-we-calculate-the-right-and-left-hand-limit-of-1-x
\mathbf{Definition} : \boxed{ \lim_{x \to a^+ } f(x) = \infty } means that for all \alpha > 0, there exists \delta > 0 such that if 0<x -a < \delta, then f(x) > \alpha \mathbf{Example} ...
Öğeler tane daha
Paylaş
Kopyala
Panoya kopyalandı
Benzer Sorunlar
\lim_{ x \rightarrow 0 } 5
\lim_{ x \rightarrow 0 } 5x
\lim_{ x \rightarrow 0 } \frac{2}{x}
\lim_{ x \rightarrow 0 } \frac{1}{x^2}
Başa dön